4
$\begingroup$

Let $k$ be a number field and $V$ a non-trivial irreducible Artin representation over $k$. Consider the associated Artin $L$-function with corresponding Euler product decomposition $L(V,s)= \prod_v L_v(V,s)$ where the product is over all non-archimedean places of $k$, for $\mathrm{re} \, s > 1$.

Then I'm fairly sure that one should have $$L(V,1) = \prod_v L_v(V,1)$$ however I can't find this in the literature. It should follow from the analogous result for Hecke $L$-functions and the usual argument via Brauer induction. Any indication where to find this in the literature would be ideal!

$\endgroup$

1 Answer 1

6
$\begingroup$

It is known that $L(V,s)$ is analytic and non-vanishing in $\Re(s)\geq 1$. Equivalently, $\log L(V,s)$ is analytic in $\Re(s)\geq 1$. Note that $\log L(V,s)$ is given by an absolutely convergent Dirichlet series $D(V,s)$ in $\Re(s)>1$. This Dirichlet series $D(V,s)$ has bounded coefficients, it is supported on prime powers, and the contribution of a given non-archimedean place $v$ equals $\log L_v(V,s)$. In particular, $$\log L(V,s)=\sum_v\log L_v(V,s),\qquad\Re(s)>1.\tag{1}$$ By Ingham's Tauberian theorem (for which Newman gave a simple proof in 1980), $D(V,s)$ converges in $\Re(s)\geq 1$. Hence, by Theorem 1.1 in Montgomery-Vaughan: Multiplicative number theory I (CUP, 2006), $D(V,s)$ is continuous in $\Re(s)\geq 1$. It follows that $$\log L(V,s)=D(V,s),\qquad \Re(s)\geq 1.\tag{2}$$ On the right-hand sides of $(1)$ and $(2)$, the contributions of prime powers that are not primes converge absolutely for $\Re(s)>1/2$, hence they are equal, while the contributions of primes are identical Dirichlet series. Hence in fact $$\log L(V,s)=\sum_v\log L_v(V,s),\qquad\Re(s)\geq 1.$$ Equivalently, $$L(V,s)=\prod_v L_v(V,s),\qquad\Re(s)\geq 1.$$

$\endgroup$

You must log in to answer this question.

Start asking to get answers

Find the answer to your question by asking.

Ask question

Explore related questions

See similar questions with these tags.